GMAT 考满分题库

Magoosh - 数学DS - 206
Judith and Quincy started with the same amount of money to invest at the beginning of 2012. Judith's investments increased by P percent in the year 2012 and then increased by Q percent in the year 2013. From the start of 2012 to the end of 2013, Quincy's investment increased by R percent. Which one of them had more money by the end of 2013?

Statement #1: P + Q < R

Statement #2:$$R-P-Q\lt\frac{PQ}{100}$$
  • AStatement 1 ALONE is sufficient to answer the question, but statement 2 alone is NOT sufficient.
  • BStatement 2 ALONE is sufficient to answer the question, but statement 1 alone is NOT sufficient.
  • CBOTH statements 1 and 2 TOGETHER are sufficient to answer the question, but NEITHER statement ALONE is sufficient.
  • DEach statement ALONE is sufficient to answer the question.
  • EStatement 1 and 2 TOGETHER are NOT sufficient to answer the question. Previous
显示答案
正确答案: B

讨论题目 或 发起提问

|

题目讨论

  • 按热度
  • 按顺序

最新提问